• We need your support!

    We are currently struggling to cover the operational costs of Xtremepapers, as a result we might have to shut this website down. Please donate if we have helped you and help make a difference in other students' lives!
    Click here to Donate Now (View Announcement)

Physics: Post your doubts here!

Messages
8,477
Reaction score
34,837
Points
698
Messages
616
Reaction score
2,961
Points
253
Yes,:) how did u get that please explain me.

look u just have to use the formula
n*(lamda)=d*sin(theta)
where n is the order (its told the second order that means 2)
d is the spacing of lines (which we have to calculate)
lamda is the wavelength
put the values in the equation
2*(590*10^-9)=d*sin(43)
solve it u wil get 1.7 :)
 
Messages
616
Reaction score
2,961
Points
253
http://papers.xtremepapers.com/CIE/Cambridge International A and AS Level/Physics (9702)/9702_w02_qp_1.pdf
Q 9
why not C?
it is t^2 so here t=(t2-t1)
Q10 why not D?
Q11 :( i thought it is C
Q14
Q21
Q32 R=V/I
so here I is infinite wont R be o??
because I=V/R and if I is infinite I=V/0??
I know there are so many but even if you help me out with one of this i will be great full
:)

Q9 the answer is D
firstly u have to find x
s=u*t+0.5a*t^2
X=0.5a*t1^2(u=o)

for the accelaration we are going to use complete hieght i.e.
x+h so
s=ut+0.5at^2 (u=0)
x+h=0.5*a*T2^2
put value of x
o.5*a*t1^2 +h= 0.5*a*t2^2
h=0.5*a*t2^2- 0.5*a*t1^2
(o.5=1/2 so on the other side it will becme 2 so it wil be 2h)
now 2h= a*t2^2 -a*t1^2
take a common
2h=a(t^2 - t1^2)
a=2h/(t2^2-t1^2)
hope u got it :/
 
Messages
1,764
Reaction score
3,472
Points
273
for 10 isnt D th answer :O??
A is the answer :(
Q9 the answer is D
firstly u have to find x
s=u*t+0.5a*t^2
X=0.5a*t1^2(u=o)

for the accelaration we are going to use complete hieght i.e.
x+h so
s=ut+0.5at^2 (u=0)
x+h=0.5*a*T2^2
put value of x
o.5*a*t1^2 +h= 0.5*a*t2^2
h=0.5*a*t2^2- 0.5*a*t1^2
(o.5=1/2 so on the other side it will becme 2 so it wil be 2h)
now 2h= a*t2^2 -a*t1^2
take a common
2h=a(t^2 - t1^2)
a=2h/(t2^2-t1^2)
hope u got it :/

thanks :)
 
Messages
616
Reaction score
2,961
Points
253
ua is going in opposite direction to ub
so ua-ub??
va is in the same direction to vb so va+vb?
why is this wrong


when the are in differentdirections one is postive the other one is negative while when the move in the same direction both are either positive or negative so when we put them in the formula we get that the velocities of the balls in different directio will be added while the ones in same direction will be subtracted
 
Messages
1,764
Reaction score
3,472
Points
273
when the are in differentdirections one is postive the other one is negative while when the move in the same direction both are either positive or negative so when we put them in the formula we get that the velocities of the balls in different directio will be added while the ones in same direction will be subtracted
either positive or negative so why subtracted it can be added too? for v and subtract for u
I am not saying that A is wrong but why is C wrong?
why cant it be like the velocities of the balls in different direction will be subtracted while the ones in same direction will be added?
 
Top